Đến nội dung

Ankh nội dung

Có 84 mục bởi Ankh (Tìm giới hạn từ 29-04-2020)



Sắp theo                Sắp xếp  

#679348 $S=\sum \dfrac{a}{ma+nb+pc}$

Đã gửi bởi Ankh on 03-05-2017 - 17:33 trong Bất đẳng thức - Cực trị

Em tìm thấy rồi, cảm ơn anh :)

Với cả hình như vấn đề này cũng được anh Cẩn giải quyết dạng tương tự bằng cách chia trường hợp trong cuốn Cauchy-Schwarz rồi thì phải.

(Mod có thể xóa cái post cảm ơn này cũng được.)




#679336 $S=\sum \dfrac{a}{ma+nb+pc}$

Đã gửi bởi Ankh on 03-05-2017 - 16:06 trong Bất đẳng thức - Cực trị

 Cho các số thực dương $a,b,c$. Xét 3 số thực dương bất kì $m,n,p$, khi đó xác định giá trị lớn nhất $S=\dfrac{a}{ma+nb+pc}+\dfrac{b}{mb+nc+pa}+\dfrac{c}{mc+na+pb}$ theo $m,n,p$.




#671713 $\sum a\sqrt{b^2+c^2}\leq \sqrt{6\sum ab}$

Đã gửi bởi Ankh on 15-02-2017 - 19:30 trong Bất đẳng thức và cực trị

m.n full giúp mk với  :icon11:  :icon11:  :icon11:  :icon11:  :icon11:

 Sau khi thuần nhất thì nó trở thành $\sum a\sqrt{b^2+c^2}\leq \sqrt{2(a^2+b^2+c^2)(ab+bc+ca)}$

 Sử dụng C-S thì $\sum a\sqrt{b^2+c^2}\leq \sqrt{(a+b+c).\sum a(b^2+c^2)}$

 Cần chứng minh $(a+b+c).\sum a(b^2+c^2)\leq \sqrt{2(a^2+b^2+c^2)(ab+bc+ca)}\Leftrightarrow \sum 2ab(a-b)^2\geq 0$




#670196 $lim2^{n}\sqrt{2-u_{n}}$

Đã gửi bởi Ankh on 28-01-2017 - 02:12 trong Dãy số - Giới hạn

Cho dãy số $(u_{n})$ thoả mãn điều kiện: $u_{1}=\frac{\sqrt{6}-\sqrt{2}}{2},u_{n+1}=\sqrt{2+u_{n}}$ với mọi $n=1,2,...$. CMR: Dãy số $(u_{n})$ có giới hạn và tìm $lim2^{n}\sqrt{2-u_{n}}$

 Đặt $u_1=2\cos \dfrac{5\pi}{12}$ và sau đó quy nạp chứng minh được $u_n=2\cos \dfrac{5\pi}{3.2^{n+1}}$, rồi sử dụng giới hạn $\lim _{x\to 0} \dfrac{\sin x}{x}=1$ là xong




#670194 $f(x+y)+f(x)f(y)=f(xy)+f(x)+f(y)$

Đã gửi bởi Ankh on 28-01-2017 - 01:12 trong Phương trình hàm

Tìm $f:R\rightarrow R$ thỏa mãn:

$f(x+y)+f(x)f(y)=f(xy)+f(x)+f(y)$ (1)

Với mọi $x,y$ thực

 Thay $y:=0$ vào (1) ta được $f(0)(f(x)-2)=0$, cho tiếp $x:=0$ suy ra $f(0)=0$ hoặc $f(0)=2$

 Với $f(0)=2$ thì suy ra $f(x)\equiv 2$

 Với $f(0)=0$, thay $x,y$ trong (1) bởi 2 suy ra $f(2)(f(2)-2)=0$

  - Nếu $f(2)=0$, thay $y=x=1$ vào (1) suy ra $f(1)=0$ hoặc $f(1)=3$

    Với $f(1)=3$, thay $y$ lần lượt bởi $1$ và $2$ vào (1) suy ra $f(x+1)+f(x)=3$ và $f(x+2)=f(2x)+f(x)$, suy ra $f\equiv 0$, vô lí

    Với $f(1)=0$, thay $y:=1$ vào (1) suy ra $f(x+1)=2f(x)$, thay tiếp $x:=x+1$ vào (1) ta có

     $f(x+y+1)+f(x+1)f(y)=f(xy+y)+f(x+1)+f(y)\Leftrightarrow f(xy+y)=2f(xy)+f(y)$, hay $f(x+y)=2f(x)+f(y)=f(x)+2f(y)$, suy ra $f$ hằng hay $f\equiv 0$

  - Nếu $f(2)=2$, thay $x,y$ trong (1) bởi 1 suy ra $f(1)=1$ hoặc $f(1)=2$ 

    Với $f(1)=2$, thay $y:=1$ vào (1) suy ra $f\equiv 2$, vô lí 

    Với $f(1)=1$, thay $y:=1$ vào (1) suy ra $f(x+1)=f(x)+1$ 

    Lại thay $x:=x+1$ vào (1) và biến đổi suy ra $f(xy+y)=f(xy)+f(y)$ hay $f(x+y)=f(x)+f(y)$, suy ra $f(xy)=f(x)f(y)$, với mọi $x,y\in \mathbb{R}$

    Từ đây suy ra $f(x)=x$ 

 Vậy có 3 nghiệm hàm thỏa mãn $f(x)\equiv 0, f(x)\equiv 2, f(x)=x,\forall x\in \mathbb{R}$




#670065 Tìm $f:\mathbb{R}\rightarrow\mathbb{R...

Đã gửi bởi Ankh on 26-01-2017 - 22:53 trong Phương trình hàm

1. PT: $f(f(x-y))+xy=f(x)-f(y)+f(x)f(y)$ (1)

Thay $x=y$ vào (1) ta được $f(x)^2=x^2+f(f(0)),\forall x\in \mathbb{R}$

 Suy ra $f(x)^2=f(-x)^2,\forall x\in \mathbb{R}$, giả sử tồn tại $\alpha \neq 0$ sao cho $f(\alpha)=f(-\alpha)$.

 Thay $y:=0$ vào (1) thì $f(f(x))=f(x)(f(0)+1)-f(0),\forall x\in \mathbb{R}$ (2)

 Thay $x:=0$ vào (1) thì $f(f(-y))=f(y)(f(0)-1)-f(0),\forall y\in \mathbb{R}$ (3)

 Từ (2) và (3) suy ra $f(-y)(f(0)+1)-f(0)=f(y)(f(0)-1)-f(0),\forall y\in \mathbb{R}$ (4)

 Thay $y:=\alpha$ vào (4) suy ra $f(\alpha)=f(0)\Rightarrow f(\alpha)^2=f(0)^2\Rightarrow \alpha ^2+f(f(0))=f(f(0))\Rightarrow \alpha =0$, vô lí.

 Do đó $f(x)=-f(-x),\forall x\in \mathbb{R}$, thay vào (4) suy ra $f(0)=0$ hoặc $f(y)\equiv 1$ (vô lí).

 Từ $f(0)=0$ suy ra $f(x)^2=x^2$ và $f(f(x))=f(x)$, từ đây suy ra $f(x)=x,\forall x\in \mathbb{R}$, thử lại thấy đúng




#664289 Cho tam giác $ABC$... Chứng minh $A,E,F,S$ đồng viên

Đã gửi bởi Ankh on 10-12-2016 - 00:09 trong Hình học

Cho tam giác $ABC$ nội tiếp $(O)$, $D$ là một điểm bất kì trên đoạn $BC$. $(ABD)$ cắt $AC$ tại $X$, $(ACD)$ cắt $AB$ tại $Y$. $(AXY)$ cắt $AD$ tại $Q$ và cắt $(O)$ tại $S$. $(ABQ)$ cắt $AC$ tại $E$, $(ACQ)$ cắt $AB$ tại $F$. Chứng minh $A,E,F,S$ đồng viên



#658655 Chứng minh bất đẳng thức

Đã gửi bởi Ankh on 21-10-2016 - 10:34 trong Bất đẳng thức - Cực trị

Cho a, b, c>0 thỏa mãn $$ab + bc + ca = 3$$. Chứng minh rằng $$2\left( {a + b + c} \right) \ge \sqrt {{a^2} + 3} + \sqrt {{b^2} + 3} + \sqrt {{c^2} + 3} \ge a + b + c + 3$$.

 Bất đẳng thức phía sau: http://diendantoanho...qrtc23geq-abc3/




#658653 $\sqrt{\frac{a^2}{a^2+b+c}}+...

Đã gửi bởi Ankh on 21-10-2016 - 10:09 trong Bất đẳng thức và cực trị

Lâu rồi mới gặp lại mà nó làm khó t.

      Bài 1: Chứng minh rằng với mọi số thực $a,b,c$, ta có:

$2(1+abc)+\sqrt{2(1+a^2)(1+b^2)(1+c^2)} \geq (1+a)(1+b)(1+c)$

      Bài 2: Chứng minh rằng với mọi $a,b,c > 0$, ta có:

$\frac{a^4}{1+a^2b}+\frac{b^4}{1+b^2c}+\frac{c^4}{1+c^2a} \geq \frac{abc(a+b+c)}{1+abc}$

      Bài 3: Cho $a,b,c$ là các số thực dương thõa mãn $a^2+b^2+c^2=3$

                 Chứng minh rằng:

$\sqrt{\frac{a^2}{a^2+b+c}}+\sqrt{\frac{b^2}{b^2+c+a}}+\sqrt{\frac{c^2}{c^2+a+b}} \leq \sqrt{3}$

 Bài 2 làm như sau

 Áp dụng bất đẳng đẳng thức Cauchy-Schwarz:

 $\sum \dfrac{a^4}{1+a^2b}=\sum \dfrac{a^4bc}{bc+a^2b^2c}\geq \dfrac{abc\left(a\sqrt{a}+b\sqrt{b}+c\sqrt{c}\right)^2}{(1+abc)(ab+bc+ca)}$

 Nên ta chỉ cần chứng minh $\left(a\sqrt{a}+b\sqrt{b}+c\sqrt{c}\right)^2\geq (a+b+c)(ab+bc+ca)$

 Mà $ab+bc+ca\leq \dfrac{1}{3}(a+b+c)^2$

 Cho nên chỉ cần chỉ ra $3\left(a\sqrt{a}+b\sqrt{b}+c\sqrt{c}\right)^2\geq (a+b+c)^3$

 Lấy căn bậc 6 hai vế thì nó tương đương $\sqrt[3]{\dfrac{\sqrt{a}^3+\sqrt{b}^3+\sqrt{c}^3}{3}}\geq \sqrt{\dfrac{\sqrt{a}^2+\sqrt{b}^2+\sqrt{c}^2}{3}}$

 Đúng theo bất đẳng thức trung bình lũy thừa




#658588 $\sum \sqrt{\frac{a^2+bc}{b^2+bc+c^2...

Đã gửi bởi Ankh on 20-10-2016 - 20:49 trong Bất đẳng thức và cực trị



Lý do nào khiến bn đặt \[B=(a^2+bc)^2(b^2+bc+c^2)(2a+b+c)^3+(b^2+ca)(c^2+ca+a^2)(2b+c+a)^3+(c^2+ab)(a^2+ab+b^2)(2c+a+b)^3\]  :wacko:  :wacko:

 Biểu thức chạy (tương tương CYH)




#658509 $\sqrt{\frac{a^2}{a^2+b+c}}+...

Đã gửi bởi Ankh on 20-10-2016 - 09:54 trong Bất đẳng thức và cực trị

theo BĐT trên thì ta sẽ đi chứng minh : 

$\sum \frac{a^4}{1+a^2b}\geq \sum \frac{a^3b}{1+ab^2}\Leftrightarrow \sum \frac{a^3(a-b)}{(1+a^2b)(1+ab^2)}\geq 0$

Ta có : $\frac{a^3(a-b)}{X}+\frac{b^3(b-c)}{Y}+\frac{c^3(c-a)}{Z}=\frac{(a-b)^2(a^2+ab+b^2)}{M}+\frac{(b^3-c^3)(a-c)}{N}\geq 0$

BĐT trên đúng nếu giả sử $a\geq b\geq c$

BĐT đc CM $☺$

Có chắc là giả sử được không? :V




#658508 $\sum \sqrt{\frac{a^2+bc}{b^2+bc+c^2...

Đã gửi bởi Ankh on 20-10-2016 - 09:52 trong Bất đẳng thức và cực trị

Còn một cách nữa là khai triển bất đẳng thức bậc 6 (cũng khá khó khăn  :mellow: )




#658507 $\sum \sqrt{\frac{a^2+bc}{b^2+bc+c^2...

Đã gửi bởi Ankh on 20-10-2016 - 09:48 trong Bất đẳng thức và cực trị

Đặt \[A=\sqrt{\dfrac{a^2+bc}{b^2+bc+c^2}}+\sqrt{\dfrac{b^2+ca}{c^2+ca+a^2}}+\sqrt{\dfrac{c^2+ab}{a^2+ab+b^2}},\]

\[B=(a^2+bc)^2(b^2+bc+c^2)(2a+b+c)^3+(b^2+ca)(c^2+ca+a^2)(2b+c+a)^3+(c^2+ab)(a^2+ab+b^2)(2c+a+b)^3\]

Áp dụng bất đẳng thức Holder ta có \[A^2B\geq \left[(a^2+bc)(2a+b+c)+(b^2+ca)(2b+c+a)+(c^2+ab)(2c+a+b)\right]^3\]

Do đó ta chỉ cần chứng minh \[\left[\sum (a^2+bc)(2a+b+c)\right]^3\geq 6\sum (a^2+bc)^2(b^2+bc+c^2)(2a+b+c)^3\]

Khai triển ta thấy bất đẳng thức trên tương đương với \[S_c(a-b)^2+S_b(c-a)^2+S_a(b-c)^2\geq 0\]

Trong đó $S_c=2(a^7+b^7)+9c(a^6+b^6)+7ab(a^5+b^5)+36abc(a^4+b^4)+9a^2b^2(a^3+b^3)+27abc^2(a^3+b^3)+60a^2b^2c(a^2+b^2)+3a^3b^3(a+b)+72a^2b^2c^2(a+b)+72a^3b^3c+6a^2b^2c^3\geq 0$

Tương tự ta cũng có $S_a,S_b\geq 0$, và do đó bất đẳng thức đã cho cần chứng minh đúng.

 Dấu "=" xảy ra khi $a=b=c$




#654369 Đề thi chọn đội tuyển quốc gia tỉnh Quảng Bình

Đã gửi bởi Ankh on 16-09-2016 - 11:02 trong Thi HSG cấp Tỉnh, Thành phố. Olympic 30-4. Đề thi và kiểm tra đội tuyển các cấp.

Bài hình ngày 1:

a, Bằng biến đổi góc ta có $DKG=FKE$ do đó $GKE=DKF=DPF$. Bằng phương tích suy ra $GDFE$ nội tiếp. Ta chứng minh được $(P,D,G)$ và $(P,F,E)$ thẳng hàng (bằng cộng góc). Từ các điều kiện trên ra có DPCM

Ai có lời giải câu b chưa vậy. Câu a khá đơn giản nhưng câu b làm tốn thời gian quá!! :(  :D

 Bạn ghi rõ đoạn chứng minh thằng hàng được không, mình cũng cộng góc mà không ra.

 P/s: Ai giải câu 5 đi




#653511 $a^x-x-1 \geq 0$

Đã gửi bởi Ankh on 09-09-2016 - 22:22 trong Phương trình - hệ phương trình - bất phương trình

Tìm $a \in \mathbb{R}$ sao cho $a^x-x-1 \geq 0$ với $\forall x \in \mathbb{R}$.

 Nếu $a\leq 1$ thì cho $x$ tiến ra vô cùng dẫn tới điều vô lí

 Khi $a>1$, bất phương trình viết lại thành $a^x\geq 1+x$

 Với $x\leq 0$, bất phương trình trên đúng với mọi $a>1$

 Với $x>0$ thì nó có thể viết lại tiếp thành $a\geq (1+x)^\frac{1}{x}$, cho $x$ tiến về $0$ suy ra $a\geq e$

 Với $a\geq e$ thì bất đẳng thức của ta đúng bằng cách kiểm tra với đạo hàm, từ đó ta có thể suy ra kết luận của bài toán




#646639 Tìm GTNN của $P=\frac{1}{\sqrt{a+b}...

Đã gửi bởi Ankh on 26-07-2016 - 21:56 trong Bất đẳng thức và cực trị

Cho $a,b,c$ là các số thực không âm thay đổi thỏa mãn điều kiện $ab+bc+ca=1$. Tìm GTNN của $P=\frac{1}{\sqrt{a+b}}+\frac{1}{\sqrt{b+c}}+\frac{1}{\sqrt{c+a}}$

 Chứng minh $f(a,b,c)\geq f\left(a+b,\dfrac{1}{a+b},0\right)$ rồi xét hàm




#646638 Số gần bình phương

Đã gửi bởi Ankh on 26-07-2016 - 21:54 trong IQ và Toán thông minh

Một số có dạng $n(n+1)$ (tích 2 số liên tiếp) gọi là số gần bình phương. Chứng minh rằng 1 số gần bình phương luôn biểu diễn được dưới dạng thương của 2 số gần bình phương khác.

 Sử dụng đẳng thức $n(n+1)(n+1)(n+2)=(n^2+2n)(n^2+2n+1)$




#646360 $\frac{1}{5} \sum_{n=1}^{5} \sqrt[n]{\prod_{i=1}^{n}...

Đã gửi bởi Ankh on 24-07-2016 - 22:49 trong Bất đẳng thức và cực trị

Lời giải cho bài toán tổng quát :)

File gửi kèm  Kedlaya.pdf   224.76K   281 Số lần tải




#646173 Inequalities From 2016 Mathematical Olympiads

Đã gửi bởi Ankh on 23-07-2016 - 21:52 trong Bất đẳng thức - Cực trị

Bài 52 (Taiwan TST). Tìm hằng số $k$ nhỏ nhất để bất đẳng thức

\[\frac{x^2y^2}{1-z}+\frac{y^2z^2}{1-x}+\frac{z^2x^2}{1-y}\leq k-3xyz,\]

luôn đúng với mọi số thực dương $x,y,z$ thỏa mãn điều kiện $x+y+z=1.$

 Áp dụng AM-GM ta có $\sum \dfrac{x^2y^2}{1-z}=\sum \dfrac{x^2y^2}{x+y}\leq \sum \dfrac{xy(x+y)}{4}$

 Nên ta sẽ có $\sum \dfrac{x^2y^2}{1-z}+3xyz\leq \dfrac{1}{4}\left(\sum xy(x+y)+12xyz\right)=\dfrac{1}{4}\left(xy+yz+zx+9xyz\right)\leq \dfrac{1}{6}$

 Hơn nữa tại $k=\dfrac{1}{6}$ thì bất đẳng thức xảy ra dấu "=" tại $x=y=z=1/3$ nên $k_{\min}=\dfrac{1}{6}$




#642678 MIN+MAX: $P=\frac{b}{a}+\frac{c}...

Đã gửi bởi Ankh on 28-06-2016 - 21:48 trong Bất đẳng thức và cực trị

Cho a,b,c dương thỏa mãn: $\frac{a}{b}+\frac{b}{c}+\frac{c}{a}=5$.

Tìm GTLN và GTNN của: $P=\frac{b}{a}+\frac{c}{b}+\frac{a}{c}$.

 Chú ý là nếu đặt $a/b=x,b/c=y,c/a=z$ thì $x+y+z=5$ và $xyz=1$, ta cần tìm $\min $ và $\max $ của $xy+yz+zx$

 Đến đây sử dụng $(x-y)^2(y-z)^2(z-x)^2\geq 0$, sau khi khai triển sẽ thu được hàm bậc 3 ẩn $xy+yz+zx$, giải bất phương trình ta tìm được cực trị :)




#641755 Chứng minh: $108-24(a^{2}b^{2}+b^{2}c^{2})\geq 27+...$

Đã gửi bởi Ankh on 22-06-2016 - 17:04 trong Bất đẳng thức và cực trị

 

Bài toán: Cho $a,b,c>0$ thoả mãn $a^{2}+b^{2}+c^{2}=3$. Hãy chứng minh:

 

               $108-24(a^{2}b^{2}+b^{2}c^{2})\geq 27+(5(a+b+c)-2(a+b+c)(ab+bc+ca)+6abc)^{2}+24c^{2}a^{2}$

 

[Chế tác từ 1 bài cũ]

 

 Sau khi đồng bậc và khai triển thì sẽ được một bất đẳng thức hiển nhiên đúng theo Muirhead :)




#641753 $\sum \frac{a^{2}b}{1+a+b}\...

Đã gửi bởi Ankh on 22-06-2016 - 16:58 trong Bất đẳng thức và cực trị

 

Bài toán:
 
Cho $a,b,c$ là các số thực không âm thỏa mãn tổng của chúng bằng 3.Chứng minh rằng:
$$(a^3b+b^3c+c^3a)(ab+bc+ca)\leq 16$$
 
-------------------------------------------------------------------------------------------------------------

 

 

Hợp lí thì lady cứ sửa! :))

 

 Chặt hơn nữa là $(a^3b+b^3c+c^3a+abc)(ab+bc+ca)\leq 16$ :)




#640578 CMR: $\frac{a+b+c}{3}\geq \sqrt[27]...

Đã gửi bởi Ankh on 15-06-2016 - 21:30 trong Bất đẳng thức và cực trị

Nguyen Ngoc Tu inequality:

Cho a, b, c là các số thực không âm thỏa mãn $$ab + bc + ca = 3$$. Chứng minh rằng: $$\frac{{a + b + c}}{3} \ge \sqrt[n]{{\frac{{{a^2} + {b^2} + {c^2}}}{3}}},\,\,\forall n \ge 6,n \in N$$.

 Chú ý là với $n\geq 6$ thì $\left(\dfrac{a+b+c}{3}\right)^n\geq \dfrac{(a+b+c)^6}{729}=\dfrac{\left(\sum a^2+2\sum ab\right)^3}{729}\geq \dfrac{3\sum a^2.81}{729}$

 

 

Taiwan, 2014:

Cho a, b, c là các số dương. Chứng minh rằng: $$3\left( {a + b + c} \right) \ge 8\sqrt[3]{{abc}} + \sqrt[3]{{\frac{{{a^3} + {b^3} + {c^3}}}{3}}}$$.

 Sử dụng bất đẳng thức Holder thì $\text{VP}^3\leq 81\left(\dfrac{a^3+b^3+c^3}{3}+8abc\right)$

 Cho nên ta chỉ cần chứng minh $27(a+b+c)^3\geq 27(a^3+b^3+c^3)\geq 648abc\Leftrightarrow (a+b)(b+c)(c+a)\geq 8abc$

 Đúng theo bất dẳng thức AM-GM




#640575 CMR $\frac{1}{a+b}+\frac{1}...

Đã gửi bởi Ankh on 15-06-2016 - 21:06 trong Bất đẳng thức và cực trị

Đóng góp cách giải khác:

Đặt: $p=a+b+c,q=ab+bc+ca,r=abc$

BĐtT cần chứng minh viết lại dưới dạng: $(p-2)(2p-1)+5r\geq 0$

* Nếu $p\geq 2$ BĐT hiển nhiên đúng.

* Nếu $\sqrt{3}\leq p\leq 2$ theo BĐT Schur bậc 3, ta có:

$r\geq \frac{p(4p-p^2)}{9}=\frac{4p-p^3}{9}$

Ta cần chứng minh $(p-2)(2p-1)+5.\frac{4p-p^3}{9}\geq 0\Leftrightarrow (2-p)(5p^2-8p+9)\geq 0$

BĐT cuối luôn đúng.

 Cách này khác gì cách của bạn hoanglong2k không :)




#640429 MAX: F=$\frac{1}{5-2ab}+\frac{1}...

Đã gửi bởi Ankh on 15-06-2016 - 09:29 trong Bất đẳng thức và cực trị

Cho a,b,c không âm thỏa mãn: $\sum a^2=3$

Tìm MAX: F=$\frac{1}{5-2ab}+\frac{1}{5-2bc}+\frac{1}{5-2ca}$

 Giả sử $b$ nằm giữa $a$ và $c$ 

 Ta có $\dfrac{3}{2}-F=\sum \left(\dfrac{1}{2}-\dfrac{1}{5-2ab}\right)=\sum \dfrac{3-2ab}{2(5-2ab)}$

 Cho neenn $3-2F=\sum \dfrac{(a-b)^2+c^2}{5-2ab}=\sum \dfrac{(a-b)^2}{5-2ab}+\sum \dfrac{c^2}{5-2ab}\geq \dfrac{4(a-c)^2+(a+b+c)^2}{15-2\sum ab}$

 Chú ý là $4(a-c)^2+(a+b+c)^2-(15-2ab-2bc-ca)=-4(b-c)(b-a)\geq 0$ nên $3-2F\geq 1$ nên $F\leq 1$

 Dấu "=" xảy ra khi $a=b=c=1$